epsilon-N definition for a limit at infinity (introduction & how to write the proof)

แชร์
ฝัง
  • เผยแพร่เมื่อ 22 ก.ย. 2024

ความคิดเห็น • 82

  • @blackpenredpen
    @blackpenredpen  ปีที่แล้ว +13

    Check out the εδ definition ultimate introduction 👉 th-cam.com/video/DdtEQk_DHQs/w-d-xo.html

  • @scottleung9587
    @scottleung9587 ปีที่แล้ว +59

    This was my absolute worst nightmare while taking Modern Analysis in college! But many thanks for walking us thru the steps of how this proof came to be.

  • @radkiller420
    @radkiller420 5 วันที่ผ่านมา

    you just explained this so well, ive watched so many videos but yours made me clearly understand this. Thankyou so much!!!

  • @shiftsync9988
    @shiftsync9988 ปีที่แล้ว +7

    This is just what I studied for an upcoming exam, great to refresh my memory. Thanks 🙏

  • @yassinelaouadi305
    @yassinelaouadi305 ปีที่แล้ว +8

    Thank you for making this demonstration waaay easier than what I learnt in school. Time to flex it on my teachers 😜

  • @dmitrybak9372
    @dmitrybak9372 ปีที่แล้ว +10

    For epsilon-delta proofs, when speaking through the proof out-loud, it becomes way more obvious for the student if the speaker says not just “for all epsilon greater than zero, …”, but “for all epsilon, no matter how small you decide to choose it, …” - i.e. emphasizing that our intent is to “make” epsilon “smaller and smaller”.
    Also “given arbitrary epsilon greater than zero” -> “given arbitrary small epsilon greater than zero”.

    • @yanceyward3689
      @yanceyward3689 ปีที่แล้ว +1

      This is an important point that often gets lost in these discussions- you can make epsilon as small as you like and will always be able to find a delta or N that satisfies the delta/N inequality, and vice versa, if the limit is in fact L. If the L is not the limit, then you will get a contradiction.

    • @nayjer2576
      @nayjer2576 6 หลายเดือนก่อน

      @@yanceyward3689to be exact, for all n > N that has to satisfy, otherwise it's a limit point

  • @lawrencejelsma8118
    @lawrencejelsma8118 ปีที่แล้ว +5

    As an engineer working on engineering precision mathematics we find the x equivalent in like time measurements all the time when 2x + 1 approximates to 2x with no significant error in precision. 3x + 4 would need a larger x to equal the same low error approximation as 2x + 1 approximate 2x replaced as 3x + 4 approximately is 3x for x >> 1 for large x. For example, x = 10^4 then the function approximates to 2x/(3x) or x in numerator and x in denominator show we are so close to 2/3 no matter if you have a ripple 1 in the numerator and a ripple 4 in the denominator compared to the 2x value / 3x value ... Not the 2 value / 3 value limit mistake in thinking.

  • @varun3282
    @varun3282 ปีที่แล้ว +4

    Good that you gave this video I'm about to start limits in my calc course

  • @haasjeoverkonijn6961
    @haasjeoverkonijn6961 6 หลายเดือนก่อน +1

    First time I understand. Great explanation. Thanks

  • @helphowdoinputusername3571
    @helphowdoinputusername3571 ปีที่แล้ว +3

    Could you do a video on how to do the proof backwards/both ways?

  • @basilbrush7878
    @basilbrush7878 6 หลายเดือนก่อน +1

    Man, you bring out the inner math genius in me, and I'm 62. I follow your logic perfectly

  • @nonentity168
    @nonentity168 11 หลายเดือนก่อน

    Damn, I appreciate your videos even more given my professor couldn't explain it properly in 3 hours.

  • @noonstar3435
    @noonstar3435 14 วันที่ผ่านมา +1

    Thank you so much these videos save my life. But do you have a video on if there's an M to find instead of an N?

    • @blackpenredpen
      @blackpenredpen  14 วันที่ผ่านมา

      @@noonstar3435 I got everything you need here: th-cam.com/video/AfrnYS5S8VE/w-d-xo.htmlsi=urptcYJbQtVVkAlS

  • @ItsMeTheUser
    @ItsMeTheUser ปีที่แล้ว +1

    Welcome back man!

  • @KaydenHimself-b3x
    @KaydenHimself-b3x ปีที่แล้ว +5

    L'hospital: it that even a question?

  • @thatomofolo452
    @thatomofolo452 ปีที่แล้ว +3

    To infinity and beyond 🚀🥳🤸♥️💫

  • @Ninja20704
    @Ninja20704 ปีที่แล้ว +5

    My teacher gave us some simple limit proof qns in my proofs class when teaching us proof by construction. And this was one of them.
    Quick question, if we wanted to prove a limit as x->-inf, do we just change to N

    • @CatchyCauchy
      @CatchyCauchy ปีที่แล้ว +1

      I think there are many ways to define it for example:
      lim x-> -inftx f(x) := lim x-> infty f(-x)
      if it exists
      Or using sequences:
      For all sequences (xn)n with xn -> -infty, then we say lim x->-infty f(x) exists if and only if lim n->infty f(x_n) exists and is the same for all sequences
      I think yours is probably aquivalent

    • @Xorven2
      @Xorven2 ปีที่แล้ว

      @Ninja20704
      Yes. A usual definition of the existence of a finite limit L of a real function f at -∞ is :
      For all Ɛ>0, it exists N

  • @WritersDigest-b8f
    @WritersDigest-b8f 7 หลายเดือนก่อน +1

    I have seen an example where the δ chosen was greater than ε. I was wondering would it not throw δ outside the ε window.
    Can the chosen δ ever be greater than ε?
    Thank u

  • @BusyBlueLion
    @BusyBlueLion 11 หลายเดือนก่อน +1

    Thank you!

  • @hritamkashyap
    @hritamkashyap 6 หลายเดือนก่อน

    Very Very Thank you sir ❤

  • @Elisha-e4m
    @Elisha-e4m 2 หลายเดือนก่อน

    beautiful

  • @ziro4614
    @ziro4614 ปีที่แล้ว +3

    I asked myself what the limit of (1+1/x)^(1/x) when x goes to infinity.
    I think it goes to 1 but I don't have a way to show it and when I asked Wolfram Aloha it says 1 but the Step-by-step solution kinda goes like e^(0/infinity) equals 1, which it say is the solution.
    Can someone help me?

    • @asifthatwouldeverhappen
      @asifthatwouldeverhappen ปีที่แล้ว

      Write the expression as exp((1/x)*ln(1+1/x))
      As x tends to infinity, 1/x tends to 0 and ln(1+1/x) also tends to 0.
      Therefore, the limit is equal to exp(0*0) = e^0 = 1
      You can then write the proof it in the epsilon N form.

    • @ziro4614
      @ziro4614 ปีที่แล้ว

      @@asifthatwouldeverhappen thanks

    • @Ninja20704
      @Ninja20704 ปีที่แล้ว +1

      A simpler way to calculate the limit is that 1/x goes to 0 as x goes to infinity. So the base approaches 1 and the power approaches 0. This is not an indeterminate form, so we can legitamately conclude that the limit is 1^0=1

  • @VaibhavSharma-zj4gk
    @VaibhavSharma-zj4gk วันที่ผ่านมา

    Doesn't, 5/9N = e(epsilon) can also be solution?

  • @Shaan_Suri
    @Shaan_Suri 8 หลายเดือนก่อน

    Hi, what do you do if you have a minus sign in the denominator, so you can't get rid of the absolute value??

  • @sadi_supercell2132
    @sadi_supercell2132 3 หลายเดือนก่อน

    How to do it with negative infinity

  • @Happy_Abe
    @Happy_Abe ปีที่แล้ว +3

    Do you have to use the max?
    What’s wrong with N being negative?

    • @spiderjerusalem4009
      @spiderjerusalem4009 ปีที่แล้ว +1

      "approaches infinity"...

    • @Happy_Abe
      @Happy_Abe ปีที่แล้ว +3

      @@spiderjerusalem4009 of course, but I’m saying if such a small N works that it’s negative should still be fine if the inequalities hold, same way N can be a random small positive real number that isn’t close to infinity

    • @stlegendff7390
      @stlegendff7390 11 หลายเดือนก่อน

      Did you get the answer to your question? Cause I have the same doubt

    • @Happy_Abe
      @Happy_Abe 11 หลายเดือนก่อน

      @@stlegendff7390 nope unfortunately not

  • @thebestchemicalelement4455
    @thebestchemicalelement4455 ปีที่แล้ว +3

    hey blackpenredpen, can you solve
    x = i^x
    as in, an infinite power tower of i's.

    • @nickulman9739
      @nickulman9739 ปีที่แล้ว +2

      i^i^i^…..=x
      x=i^x
      x=e^(ln(i^x))
      x=e^(xln(i))
      note: (ln(i)= (pi*i)/2 for the complex logarithm principal branch, this can be observed also through rulers identity e^(pi*i) = -1 as putting both sides to the power of 1/2 results in e^((pi*i)/2)=i.)
      x=e^(x*i*pi/2)
      x/(e^(x*i*pi/2)) = 1
      x(e^(-x*i*pi/2))=1
      x*(-i*pi/2)*(e^(-x*i*pi/2))=(-i*pi/2)
      W(x*(-i*pi/2)*(e^(-x*i*pi/2)))=W(-i*pi/2)
      -x*i*pi/2=W(-i*pi/2)
      x=(2i/pi)*W(-i*pi/2)
      Sorry for bad formatting, am commenting on phone.

  • @teddy05p
    @teddy05p ปีที่แล้ว

    Lovely video, though I would like to point out a correction that for epsilon-N it should be stated as a defined sequence lets say a_n, so rather a set of a sequence than as you said a function:) But both work fine I guess! Ty for the videos!

  • @IloveTwinkies412
    @IloveTwinkies412 11 หลายเดือนก่อน

    respect from bmstu

  • @neekunjchaturvedi
    @neekunjchaturvedi ปีที่แล้ว +4

    Can we not do taking 1/x tend to 0 if x tends to infinity

    • @sivanaidoo5602
      @sivanaidoo5602 ปีที่แล้ว +1

      Not calculate but prove it is 2/3. So the limit definition is applicable.

    • @Ninja20704
      @Ninja20704 ปีที่แล้ว +1

      You can use this definition to prove that.
      Given any epsilon>0
      Choose N = 1/epsilon
      Suppose x>N.
      Check |1/x-0|=1/x (x>0 so 1/x>0)

    • @neekunjchaturvedi
      @neekunjchaturvedi ปีที่แล้ว +1

      @@Ninja20704 thanks brother

  • @etcthedeimos
    @etcthedeimos ปีที่แล้ว

    Nice video

  • @MikiMiusA
    @MikiMiusA 9 หลายเดือนก่อน

    Don't we need to write the conclusion at the end?

  • @mihailangelov6
    @mihailangelov6 ปีที่แล้ว

    When I will have access to America?

  • @naxnusternann332
    @naxnusternann332 ปีที่แล้ว +13

    Are you not allowed to use L’Hopital for a prove?

    • @SimsHacks
      @SimsHacks ปีที่แล้ว +20

      Of course not. That's not a proof, that's a calculation technique. Furthemore, these easy limits are usually covered before derivatives.

    • @photophone5574
      @photophone5574 ปีที่แล้ว +2

      @@SimsHacksI think you are right on it coming before derivatives, but calculation techniques do work in proofs. Calculation techniques in proofs is just called algebra

    • @SimsHacks
      @SimsHacks ปีที่แล้ว

      @@photophone5574 it's allowed if you had previously proved l'Hospital's rule. Which I don't think is the case.

    • @zarkuz
      @zarkuz ปีที่แล้ว +2

      No you definably can’t due to the lack of proof and evidence

    • @spiderjerusalem4009
      @spiderjerusalem4009 ปีที่แล้ว +3

      please stop bringing this overly used method up, notably whose validity is beyond your question. Utter tiresome.🙄

  • @ceo1OO
    @ceo1OO ปีที่แล้ว +2

    The blackPenredPen guy is putting on weight: ... he must be eating alot of general Tso's chicken... lol.. it's alright, I eat general Tso's chicken also... 😎

  • @ussr8410
    @ussr8410 ปีที่แล้ว +5

    Use L'Hopital's rule what's the problem bruh😅

    • @tiangong6616
      @tiangong6616 ปีที่แล้ว +4

      But this is about how to write a formal rigorous proof, not obtain the final answer, L'Hopital's rule is more like a calculation technique

    • @mathe-magics8451
      @mathe-magics8451 13 วันที่ผ่านมา +1

      Abhi aap chote ho

  • @Douae1111
    @Douae1111 10 หลายเดือนก่อน

    3:36

  • @JB-ym4up
    @JB-ym4up ปีที่แล้ว +1

    I just multiply the top and bottom by (1/x).

  • @AdityaKumar-gv4dj
    @AdityaKumar-gv4dj ปีที่แล้ว +1

    Please solve this question for me:
    Let f(x)=(x²+x+1)/(x²-x+1), then the largest value of f(x) for all x belongs to [-1,3].
    Can you please teach me how to apply x's bound over such f(x)?

  • @SamsungA04e-dp7kj
    @SamsungA04e-dp7kj 6 หลายเดือนก่อน

    dalil d La Hospital
    Undefined : Proof Theory Graph Trayektory

  • @jotajaviergonzalezgarcia7504
    @jotajaviergonzalezgarcia7504 ปีที่แล้ว

    Lhlpital 3secs

  • @broytingaravsol
    @broytingaravsol ปีที่แล้ว +1

    still appealing to the version in English

  • @ingramr88
    @ingramr88 ปีที่แล้ว

    If the first thing you comment it L’Hospitals, we already know that you’re not going to get out of this video what you need to get out of this video.
    It’s very telling.
    Come back after a discrete math class then look at this video lol

  • @siamchowdhury3987
    @siamchowdhury3987 ปีที่แล้ว

    This chanels name starts with my favourite k pop bands name.

    • @picup30296
      @picup30296 ปีที่แล้ว +4

      the one who copy la campanella?

  • @willthecat3861
    @willthecat3861 ปีที่แล้ว +2

    My cat, and I, have watched a lot of BR's proof vids. My cat is much better than me at giving all the proofs... just copies BR's proof. I don't think my cat understands math... do you?

  • @cagryldz4089
    @cagryldz4089 หลายเดือนก่อน

    hayatımda bu kadar rezalet bir anlatıma şahit olmadıım.

  • @deveshkumardhayanidhi258
    @deveshkumardhayanidhi258 หลายเดือนก่อน

    0:16 😂